2
$\begingroup$

Suppose you have a number $$ N = p_1^{e_1}p_2^{e_2}\cdots p_k^{e_k} $$ and are looking for the largest divisor $d|N$ such that $d^2<N$ (that is, A060775$(N)$.) How can I efficiently find this $d$?

If $N$ has a small number of divisors, you can just iterate through them all (keeping the best at each step). If $e_i$ is large for some $i$, then you can search the divisors $d$ of $N/p_i^{e_i}$ less than $\sqrt{N}$ and find $$ e=\min\left(\left\lfloor\frac{\log(\lfloor\sqrt{N-1}\rfloor/d)}{\log p_i}\right\rfloor, e_i\right) $$

(this would be

s=sqrtint(N-1);
f=factor(N);
e=min(logint(s\d, f[i,1]), f[i,2])

in PARI/GP; presumably lots of languages have efficient implementations)

giving $p_i^ed$ as the number to test. But for the general case where $k$ is large neither is practical. Is there some method which takes significantly fewer than $\tau(N)$ steps? You can assume that the factorization is given.

$\endgroup$
7
  • 3
    $\begingroup$ You want $\sum f_i\log p_i$ to be less than but close to $(1/2)\log N$, with $f_i$ integers, $0\le f_i\le e_i$. This is a bin-packing problem, which leads me to suspect there's no especially efficient algorithm for it. $\endgroup$ Mar 18, 2022 at 2:28
  • $\begingroup$ Yes, especially since the numbers are large, the well known polynomial time algorithm for the subset-sum problem (cf Wikipedia) is not very efficient, since you'll have to take the epsilon very small to get the correct result. $\endgroup$
    – Max
    Mar 22, 2022 at 17:03
  • 1
    $\begingroup$ For the square free case, Broardhurst, Carmody, Gerbicz et al have discussed this back in 2011 on the primenumbers list and found an interesting implementation based on finding the smallest a such that a²-4N is a square. It still becomes impractical around primorial(60). Maybe that a can be found or constructed more efficiently... $\endgroup$
    – Max
    Mar 22, 2022 at 17:20
  • $\begingroup$ @Max That’s definitely worth an answer! $\endgroup$
    – Charles
    Mar 22, 2022 at 17:21
  • $\begingroup$ In another SE question it is proved that a < b are central divisors of ab iff sqrt(a) > sqrt(b) - 1. Could this be used to make the search of the f_i more efficient? $\endgroup$
    – Max
    Mar 22, 2022 at 17:27

2 Answers 2

6
$\begingroup$

I believe the Schroeppel & Shamir algorithm [1] can be adapted for use here, with time something like $O\left(\sqrt{\tau(n)}\omega(n)\right)$ and space something like $O\left(\sqrt[4]{\tau(n)}\right)$. The basic idea is splitting the prime powers into four sets with roughly the same number of divisors each, listing the divisors, and generating divisors from two pairs of two sets dynamically using min-heaps, and combining as expected.

[1] Richard Schroeppel and Adi Shamir, A $T = O(2^{n/2})$, $S = O(2^{n/4})$ algorithm for certain NP-complete problems, SIAM Journal on Computing, Vol. 10, Iss. 310, pp. 456–464.

$\endgroup$
3
  • $\begingroup$ Please let me know when you succeed in implementing this! $\endgroup$
    – Max
    Mar 22, 2022 at 17:09
  • $\begingroup$ The breakthrough in that paper was memory reduction, which was a big issue back in 1980s. Nowadays one can easily rely on $O(\sqrt{m})$ memory for $m\leq 2^{60}$ or so, and further memory reduction with related computational overhead is needed only for larger values, for which however the time complexity will kick in as a major limitation. $\endgroup$ Apr 7, 2022 at 13:57
  • $\begingroup$ @MaxAlekseyev Absolutely—if your problem is small enough just split it in two and avoid the hassle of this method. But this algorithm greatly expanded the possible range of problems to be solved. $\endgroup$
    – Charles
    Apr 7, 2022 at 20:22
5
$\begingroup$

I believe the problem is actually a special case of a knapsack problem --- for $\vec c = (\log p_1,\dots,\log p_k)$, you have to maximize $\langle \vec x, \vec c\rangle$ subject to the constraints

  1. $\forall i : 0\leq \vec x_i \leq \vec e_i$, and
  2. $\langle \vec x, \vec c\rangle < \langle \vec e, \vec c\rangle/2$.

Written in this way, it is simple to see that this is a "special case" of a "bounded knapsack problem" (the bounded knapsack problem allows the constraint labelled 2 above to be of the form $\langle \vec x, \vec c\rangle < W$ for arbitrary $W$). This is to say that it is at least closely related to an NP hard problem, but the above does not allow us to conclude that it is NP hard.

If you are fine with efficient approximate answers, being an instance of the bounded knapsack problem is good. This is because the bounded knapsack problem admits an FPTAS, so for any $\epsilon > 0$, there is an algorithm that finds solutions within an $(1-\epsilon)$ factor of optimal in time $\mathsf{poly}(k, 1/\epsilon)$, so quite efficiently. If you let $\vec x_{opt}$ be the optimal solution, this will approximation algorithm will return some $\vec x$ such that

$$(1-\epsilon)\langle \vec x_{opt}, \vec c\rangle \leq \langle \vec x, \vec c\rangle \leq \langle \vec x_{opt}, \vec c\rangle.$$ Rewriting back in terms of the inital problem, if $d_{opt} = \exp(\langle \vec x_{opt}, \vec c\rangle)$ is the optimal divisor, we will get a divisor $d$ such that

$$d_{opt}^{1-\epsilon} \leq d\leq d_{opt}.$$

This is already good enough to get an extremely accurate approximation to your problem. In particular, choosing $\epsilon = \frac{1}{\log_2 N}$ (which will still yield a poly-time computation), we get that $\left(N/2\right)^{\frac{\log_2 d_{opt}}{\log_2 N}}$. Under the (to me reasonable) assumption that $\log_2 d_{opt}\approx \frac{1}{2}\log_2 N$, we therefore get a divisor lower-bounded by the quantity $\sqrt{N/2}$, meaning nearly optimal. I don't know if a high-quality approximation is sufficient for your purposes though.

$\endgroup$
4
  • 3
    $\begingroup$ This is great information, thank you! I am looking for an exact answer, though. $\endgroup$
    – Charles
    Mar 18, 2022 at 3:16
  • 2
    $\begingroup$ I have tested this approach to compute this sequence (upper/lower central divisior) for the primorials, it's not practicable beyond ~25 or 30. The epsilon must be extremely small to be only close to the exact result, and computation time explodes. $\endgroup$
    – Max
    Mar 22, 2022 at 17:08
  • $\begingroup$ A related question about practical issues with ILP solvers for this problem: mathoverflow.net/q/419722 $\endgroup$ Apr 6, 2022 at 13:06
  • $\begingroup$ @Mark These two Mathematica programs: chat.stackexchange.com/transcript/message/63872730#63872730 find the divisors of 9 and 12 respectively, as subsets of the solutions to a knapsack problem. $\endgroup$ Jun 28, 2023 at 14:45

Your Answer

By clicking “Post Your Answer”, you agree to our terms of service and acknowledge you have read our privacy policy.

Not the answer you're looking for? Browse other questions tagged or ask your own question.